You are on page 1of 56

June 2007 Exam Solutions

Section I: Logic Games

GAME #1: Numbered Ordering

Conceptualize the Game


This game requires that we order a set of five elements: 0, 1, 2, 3, and 4. This is a unique situation in that
we are asked to order elements that are actually numbers instead of letters. This can cause some confusion
if we're not careful. We must be sure not to confuse the elements to be ordered (0, 1, 2, 3, 4) with the
positions in which they will be placed (1st, 2nd, 3rd, 4th, 5th). Given the fact that each code must be a
"five-digit" code, we can start by creating a Number Line to represent the five positions. We'll put the
elements to be ordered off to the side:

0
1
2
3 1st 2nd 3r 4th 5th
4

The second constraint gives us important information: Each digit occurs exactly once in any code. This
means that any valid code will use each one of the five elements (0, 1, 2, 3, 4) once and only once (no
repeats), and since we're dealing with a five-digit code, each slot above must be filled (no slots will be left
empty). This may seem obvious for this particular game, but it's always important to "conceptualize" the
game in this way before you begin. To help you conceptualize, always ask yourself the following
questions: Will any of the elements be used more than once? Will any of the elements go unused? Will
any of the slots be left unfilled? If you can get all of this straight before you begin, your task will be made
much easier.

Setup
Let's see how we can use the constraints to make some inferences about the product codes. We know that
the second digit has a value that is twice the value of the first digit. This constraint limits the possibilities
for the 1st and 2nd slots. The only way that the second digit can be twice the first is if we have 1 and 2 or
2 and 4 in the first two slots. We'll put a "bubble" around the remaining three digits in each case to
indicate that these digits will occupy the last three slots, though in an uncertain order. Here are the two
possible frames:

Frame Frame

0, 3, 4 0, 1, 3
1 2 2 4

1st 2nd 3r 4th 5th 1st 2nd 3r 4th 5th


We also know that the value of the third digit must be less than the value of the last digit. In Frame #1,
this means 4 can't go in the 3rd slot and 0 can't go in the 5th slot. In Frame #2, this means 3 can't go in the
3rd slot and 0 can't go in the fifth slot. We'll indicate this by putting some cross-outs underneath:

Frame Frame

0, 3, 4 0, 1, 3
1 2 2 4

1st 2nd 3r 4th 5th 1st 2nd 3r 4th 5th


4 0 3 0

The Questions
At this point we’re ready to apply our diagrams to the questions.

1. (A)
Question Type: Conditional
This question provides us with a piece of conditional ("If") information that pertains only to this question:
the last digit is 1. What inferences can we make from this new information? Looking at our two frames
above, we see that 1 can be last only in frame #2. In fact, if 1 is last in frame #2, then the product code
would have to be: 2 4 0 3 1.

(A) Yes! We must have a 2 in the 1st slot. We can choose this and move on.

2. (C)
Question Type: Unconditional
This question does not provide us with any conditional ("If") rule in the question. It asks which of the answers "must
be true." Thus, if we can prove an answer to be untrue in at least one case, that answer is wrong.

(A) The digit 1 does not always have to appear before the digit 2. In any frame #2 arrangement, the 1 will appear
after the 2, not before it. Eliminate it.
(B) The digit 1 does not always have to appear before the digit 3. In frame #2, we could have 2 4 0 3 1, which puts
the 1 after the 3, not before it. Eliminate it.
(C) Yes! This must be true. The 2 must come before the 3 in any arrangement.

3. (C)
Question Type: Conditional
This question provides us with a piece of conditional ("If") information that pertains only to this question: the third
digit is NOT 0. What inferences can we make from this information? In frame #1, if 0 is NOT in the 3rd slot it must
go in the 4th slot (remember, 0 can’t go 5th). We’d have: 1 2 3 0 4. We can use the same logic in frame #2. The 0
must go in slot 4, which means we’d have: 2 4 1 0 3. These are the only two possibilities. We’re looking for an
answer that “must be true.”
(A) The second digit could be 2, but it need not be 2. Again, frame #2 would give us 2 4 1 0 3, which puts 2 in the
first position. Eliminate it.
(B) The third digit could be 3, but it need not be 3. Frame #2 would give us 2 4 1 0 3, which puts 3 in the fifth
position. Eliminate it.
(C) Yes! The 0 is in the 4th slot in both cases: 1 2 3 0 4 and 2 4 1 0 3. We can move on without checking (D) and
(E).

Frame Frame

0, 3, 4 0, 1, 3
1 2 2 4

1st 2nd 3r 4th 5th 1st 2nd 3r 4th 5th


4 0 3 0

4. (E)
Question Type: Unconditional
This question provides us with no conditional ("If") information. This is a tricky EXCEPT question. It's
important that we anticipate the results before we begin: four of the answers could be valid in one or more
cases, and one answer will never be valid. We're looking for the one answer that can never be valid.

(A) This could be true. 0 and 1 could be third and fourth. Frame #2: 2 4 0 1 3. Eliminate it.
(B) This could be true. 0 and 3 could be third and fourth. Frame #1: 1 2 0 3 4. There’s another case in
frame #2 that puts 0 and 3 in the third and fourth slots, but all we need is one case to prove it could be
true. Eliminate it.
(C) This could be true. 1 and 0 could be third and fourth. Frame #2: 2 4 1 0 3. Eliminate it.
(D) This could be true. 3 and 0 could be third and fourth. Frame #1: 1 2 3 0 4. Eliminate it.
(E) This is impossible. If 3 and 4 are in the third and fourth slots (frame #1), that would put 0 in slot 5.
We already know this can’t be true.

5. (E)
Question Type: Unconditional
This question provides us with no conditional ("If") information. We're looking for an answer that "must be true" in
all cases. Thus, if we can find at least one case that proves an answer untrue, we can eliminate that answer.

(A) There doesn’t have to be exactly one digit between the 0 and 1. For example, frame #2 could give us: 2 4 0 1 3.
This puts no digits between the 0 and 1. Eliminate it.
(B) There doesn’t have to be exactly one digit between the 1 and 2. Any arrangement from frame #1 puts the 1 and
the 2 adjacent to each other. Eliminate it.
(C) We can actually get more than two digits between the 1 and the 3. From frame #1 we could have: 1 2 0 4 3. This
puts three digits between the 1 and 3. Eliminate it.
(D) We can actually get more than two digits between the 2 and the 3. From frame #2 we could have: 2 4 0 1 3. This
puts three digits between the 1 and 3. Eliminate it.
(E) Yes! In no case can we get more than two digits separating the 2 and 4. This must be true.

NOTE: Before tackling the questions, you may have decided to list out all the possible product codes for this game.
We didn’t take this approach, but it is certainly possible (as it turns out, there are only six possible product codes
that meet the constraints). This is a VERY unique game in this regard. Generally, it is NOT a good time investment
to go down the road of listing out all the possibilities. In fact, most of the time, you’ll get partway there and realize
that there are more possibilities than you anticipated. While there are some very rare cases for which this will work,
you should always be able to do the game without listing all the possibilities (as we did above).

GAME #2: Open Assignment

Conceptualize the Game


Three films (G, H, and L) will be shown on three days: Thursday, Friday, and Saturday. We know that at least one
film will be shown on each day, but we could potentially have all three films on any given day. We can represent
this as follows:

GHL

Th Fri Sat

We have a maximum of three slots for each day (since we could have at most three films on any given
day), and we’ve boxed one slot for each day to indicate that at least one film must be shown on each day
(the boxes must be filled with something). So, let’s be sure we’re clear on how this will work. Will each
of the three films be used? Yes. The scenario says that each film will be shown at least once. Will all the
slots be filled? No, not necessarily. We know that a minimum of one and a maximum of three slots will
be filled for each day. In other words, some slots may be left empty. Will any of the films be used more
than once? Possibly, but not on the same day. In fact, it is likely that we’ll use the films more than once.
Remember to always conceptualize the game in this way before you begin.

There is one other important piece of “conceptualization” information that we need to pick up from the
constraints. There is an ordering component to this game! Notice that the constraints mention things like
“no film is shown after it.” Thus, in addition to tracking how many films are shown on a given day, we’ll
need to track which films are shown before and after other films on a given day. There’s an easy way to
account for this:

3rd:

2nd:
GHL

1st:
Th Fri Sat
Setup
Let’s diagram the constraints. The first constraint tells us that H is shown on Thursday, and that no other
film is shown after H on Thursday. Don’t be fooled into thinking that H must be the only film shown on
Thursday. It could be the only film shown, but there are other possibilities as well. In fact, all three films
could be shown on Thursday (with H in the 3rd spot). We’ll diagram this constraint as follows:

H
(last)

3rd:

2nd:
GHL

1st:
Th Fri Sat

The next constraint tells us that either G or L (but not both) is shown last on Friday. Note that if G or L,
but not both, is shown on Friday, then we can have at most two films shown on Friday. Thus, we’ll cross
out the third slot on Friday:

H G or
(last) (last)

3rd:

2nd:
GHL

1st:
Th Fri Sat

The last constraint tells us that either G or H (but not both) is shown last on Saturday. Again, if G or H,
but not both, is shown on Saturday, then we can have at most two films shown on Saturday. Thus, we’ll
cross out the third slot on Saturday:
H G or G or H
(last) (last) (last)

3rd:

2nd:
GHL

1st:
Th Fri Sat

The Questions
Now that we’ve got the constraints translated we can move on to the questions.

6. (C)
Question Type: Unconditional (Orientation)
Orientation questions ask us to choose a complete and accurate arrangement of the elements (film
showings, in this case). Orientation questions often lead off the set of questions. They can be solved by
working through the constraints one by one and eliminating any choice that violates the constraint. Start
with the constraints, NOT the answer choices.

We’ll start with the first constraint: H is shown on Thursday, and no other film comes after it on
Thursday. Answer (D) violates this constraint. Eliminate it.

The second constraint: G or L, but not both, is shown on Friday, and nothing comes after it. Answer (B)
has both G and L on Friday. Eliminate it. Answer (E) has just L on Friday, but with H after it. Eliminate
(E).

The final constraint: G or H, but not both, is shown on Saturday, and nothing comes after it. All the
remaining answer choices conform to this constraint.

We still have (A) and (C) remaining, so let’s take a closer look. Remember that each film must be shown
at least once. Answer (A) leaves out G. Eliminate it.

7. (A)
Question Type: Unconditional
This question does not give us any conditional (“If”) information. The wording in the question can be
tricky, so let’s be sure we anticipate the answer choices: four of the choices could be true in at least one
case, and one of the choices CANNOT be true in any case. We’re looking for the one that can’t be true.
(A) Bingo! Looking at our diagram, we know that H must be the last film shown on Thursday, and it
could be the last film shown on Saturday, but it CANNOT be the last film shown on Friday. Thus, H
cannot be the last film shown on each day of the festival. We can choose (A) and move on.

H G or G or H
(last) (last) (last)

3rd:

2nd:
GHL

1st:
Th Fri Sat

8. (D)
Question Type: Conditional
The question gives us a piece of extra, conditional (“If”) information that will apply to this question only:
L is never shown again once G is shown. Given this condition, we’re looking for the maximum number of
film showings. We already know that Friday and Saturday are limited to two showings each. So, even
without the new condition, we can have a max of seven showings. Let’s see if we can make the condition
work with seven showings.

To start, we’d need to fill all of Thursday’s slots. We know that H has to be last on Thursday, and the new
condition says that L would have to come before G. We can do this: L first, G second, H third.

We must keep in mind, however, that we can’t use any more L’s (since G has been shown). So, for Friday
we’d have to do: H first, G second (since G would need to be last). So far so good! We have a problem
with Saturday, though. We can’t use any more L’s, and we can’t show both G and H on Saturday. Thus,
we can only show one film on Saturday, G or H.

Given the constraint, we can’t show seven films, but we just proved that six is possible.

9. (E)
Question Type: Conditional
Tough question! This question gives us multiple pieces of extra, conditional (“If”) information that will
apply to this question only: G is shown three times, H twice, and L once. Given these conditions, we’re
looking for the answer that “must be true” in all cases.

The first thing we want to do is use our diagram and the additional information to create inferences.
First, if G is shown on all three days, then H can’t be shown on Saturday (remember, either G or H, but
not both, is shown on Saturday).

Next, if H is shown twice, and if H is NOT shown on Saturday, then H must be shown on Thursday and
Friday. Remember that G is shown on all three days, so Friday must have H first and G second.

If we do a good job of following the inference chain, we’ll often be led directly to the correct answer.
Knowing that H and G are both shown on Friday seems like a big inference. Let’s check the answers to
see if one matches. Sure enough, (E)’s the one. On conditional questions, follow the inference chain
before you start a trial and error approach! You’ll most definitely save yourself a lot of time.

H G or G or H
(last) (last) (last)

3rd:

2nd:
GHL

1st:
Th Fri Sat

10. (D)
Question Type: Conditional
This question gives us multiple pieces of extra, conditional (“If”) information that will apply to this
question only: L is shown three times, H twice, and G once. Given these conditions, we’re looking for a
complete and accurate list of the films that could be the first film shown on Thursday.

Again, we want to start by using the additional information to create an inference chain.

First, if L is shown three times, then L must be shown on Thursday. We already know that H must be
shown on Thursday as well, and that nothing can follow H on Thursday (H must be last). So, L must be
shown before H on Thursday. Thus, H CANNOT be shown first on Thursday. We can eliminate any
answer that includes H. Eliminate (A), (C), and (E).

We’re down to (B) and (D). Both contain L, so we know that L can indeed be shown first on Thursday.
The real question is whether G can be shown first. Let’s try it.

If G is shown first on Thursday, we’d have G first, L second (L must be shown on all three days), and H
third (H must be shown on Thursday, and H must be last):
H G or G or H
(last) (last) (last)

3rd: H

2nd: L
GHL

G
1st:
Th Fri Sat

Remember that an additional condition for this question is that we can only use one G. Thus, H must go
last on Saturday (either H or G goes last on Saturday). We know that L has to be shown on all three days,
so Saturday would be: L first, H second:

H G or G or H
(last) (last) (last)

3rd: H

2nd: L H
GHL

G L
1st:
Th Fri Sat

Now we’ve used up our one G and our two H’s (remember that an additional condition for this question is
that we can only use two H’s). So, the only thing that can go on Friday is L. Here’s the complete picture:
H G or G or H
(last) (last) (last)

3rd: H

2nd: L H
GHL

G L L
1st:
Th Fri Sat

We’ve just shown that G can go first on Thursday, so the correct answer is (D).

GAME #3: Numbered Ordering

Conceptualize the Game


We have seven consecutive trips scheduled to four destinations: G, J, M, and T. The number line is the
best diagramming approach for this question:

G
J
1 2 3 4 5 6 7 M
T

Conceptualization of this game is very important. Will all the slots be filled? Yes. The scenario says that
there will be seven voyages. Will all the letters be used? Yes. The scenario says that each destination will
be scheduled for at least one of the weeks. Will we have any repeats? Yes! If we have only four letters,
and we need to fill seven slots, we need to use at least one of the letters more than once. If you don’t
realize this before you begin, it will be difficult to succeed on this game.

Setup
Let’s diagram our constraints. The first constraint says that J will not be the destination in week 4 and that
T will be the destination in week 7:

G
T J
1 2 3 4 5 6 7 M
T
J
The third constraint says that we’ll have exactly two M’s (no more, no less), and that we’ll have at least
one G between those two M’s. We’ll note this in symbol form next to our diagram:

exactly 2M
G
T
M G M J
1 2 3 4 5 6 7 M
T
J

The M – G – M notation is what we call Relative Ordering notation; it has a very specific meaning. It
means that we must have an M at some time before the G (though we’re not quite sure how many slots
before the G), and an M at some time after the G (though we’re not sure exactly how many slots after the
G). In other words, other letters could come between them. Do NOT confuse this with MGM, which
would mean that the three letters lie adjacent to each other.

The next constraint tells us that G will come immediately before any J. So, whenever we have a trip to J,
we MUST have a trip to G in the immediately preceding week. Note that if this is the case then we
CANNOT have a trip to J in week 1, as there would be no room to put a G before it. We’ll note this with
a cross-out below week 1:

exactly 2M
G
T
M G M J
1 2 3 4 5 6 7 M
J GJ T
J J

Now, we must be very careful with this last constraint. We know then when we have a J, we must have a
G right before it (as indicated by the arrow symbol). However, this does NOT mean that every G must be
followed by a J. Think of it this way: J is the trigger. Whenever we see J, that triggers a G before it. G,
however, is NOT a trigger. So, seeing a G doesn’t necessarily trigger anything. For those of you who have
already learned about conditional logic, this probably makes perfect sense.

The last constraint tells us that no destination will be scheduled for consecutive weeks. In other words, we
can’t have two of the same letters next to each other. We’ll symbolize this using a cross-out of two
generic X’s next to each other. Note that if we can’t have two like letters adjacent to each other, we know
that we can’t put a T in slot 6. We’ll notate this with a cross-out:

exactly 2M
G
T
M G M J
1 2 3 4 5 6 7 M
J GJ T
J J T
XX

The Questions
Our setup is complete and we’re ready for the questions.

11. (A)
Question Type: Unconditional (Orientation)
Orientation questions ask us to choose a complete and accurate arrangement of the elements. Orientation
questions often lead off the set of questions. They can be solved by working through the constraints one
by one and eliminating any choice that violates the constraint. Start with the constraints, NOT the answer
choices.

First constraint: J can’t go in slot 4. Eliminate (D).

Second constraint: T must go in slot 7. Eliminate (E).

Third constraint: Exactly two M’s, with a G in between. (B) has no G between the M’s. Eliminate it.

Fourth constraint: G comes immediately before any J. (C) has a J in slot 1! Eliminate it. (A) is left.

exactly 2M
G
T
M G M J
1 2 3 4 5 6 7 M
J GJ T
J J T
XX

12. (A)
Question Type: Unconditional
The question doesn’t give us any additional “If” constraint. The wording can be tricky here, so let’s be
sure we anticipate the answer choices: four of the answers could be true in at least one case, and one
answer can never be true. We want to find the one that can never be true.

(A) We already know that T CANNOT go in week 6! Remember, this is one of the first inferences we
made (and notated with a crossed out T below week 6). We can choose (A) and move on.

13. (D)
Question Type: Conditional
This question gives us an extra piece of conditional information that we need to apply to this question
only: T goes in week 5. Let’s create a special diagram for this question, putting T in slot 5 (we don’t want
to write on the original diagram because we’ll need that for future questions):

T T

1 2 3 4 5 6 7
J J T

As we’ve mentioned previously, whenever we get an additional piece of information, we want to see if
we can generate any inferences from it. In this case, inferences are tough to find, so we’ll move to the
answer choices. Unfortunately, this question is going to require some trial and error. In this case, we’re
looking for an answer that could be true in at least one case. These are tougher to find.

(A) Can we put a T in slot 1? Let’s jot it down quickly to see what happens: T _ _ _ T _ T. Remember,
we also need to use two M’s, a G, and a J. These four letters would need to fill the remaining four spots.
Also note that the G needs to come right before the only J (one of our original constraints), and that the G
must come between the two M’s. The only way to do this is: T M G J T M T. But wait! This puts a J in
slot 4. This can’t happen. Since we’re looking for an answer that could be true, we can eliminate (A).

(B) Can we put an M in slot 2? We’d have: _ M _ _ T _ T. Same problem as before. We need a J in there
somewhere, and we need a G right before it. The only available space for a GJ is slot 3 and slot 4. We
can’t put a J in slot 4, so we can eliminate (B).

(C) Can we put a G in slot 3? We’d have: _ _ G _ T _ T. Our J can’t go in slot 4, so it’d have to go in slot
2 (in order to get a G before it): G J G _ T _ T. This leaves our two M’s: G J G M T M T. But this means
we don’t have a G between the two M’s! Eliminate it.

(D) Can we put an M in slot 4? We’d have: _ _ _ M T _ T. We could put J in slot 3, and then a G before
it: _ G J M T _ T. This means we’d need the other M in slot 1 (in order to get a G between the two M’s),
and we could put a G in slot 5: M G J M T G T. This works! (D) is the answer. We can move on.

exactly 2M
G
T
M G M J
1 2 3 4 5 6 7 M
J GJ T
J J T
XX

14. (E)
Question Type: Conditional
We have a lot of new conditional “If” information for this question: G in slot 1 and J in slot 5. Well, if J is
in slot 5, we know that G must be in slot 4 as well (whenever we have J we must have G right before it).
We’ll quickly draw a temporary diagram to include all of this information:
G G J T

1 2 3 4 5 6 7
J J T

We’ve already made one inference – can we make any others? Well, remember that we have to have two
M’s on either side of a G somewhere in our ordering. We can’t put M’s around the G in slot 1, so we must
have M’s somewhere to the right and left of the G in slot 4. The only way to do this is to have an M in
slot 6. The question is asking for something that “must be true” given the extra constraints. Well, we just
discovered that we must have an M in slot 6. Let’s look to see if this inference shows up among the
choices. Indeed it does in (E).

15. (A)
Question Type: Conditional
Here we’re given the conditional information that G is in slot 1 and T is in slot 2:

G T T

1 2 3 4 5 6 7
J J T

Can we derive any inferences from this information? Yes! We know we must have our M – G – M in
there somewhere, and that we must have a J in there somewhere, too. The only way to do this is by
putting the M’s in 3 and 6 and the GJ between: G T M G J M T. Now, we’ll simply look for an answer
that matches. We’re looking for an answer that “must be true.” Indeed M must be placed in slot 3. (A) is
the answer.

exactly 2M
G
T
M G M J
1 2 3 4 5 6 7 M
J GJ T
J J T
XX
16. (A)
Question Type: Conditional
This question gives us a conditional piece of information: M in week 3:

M T

1 2 3 4 5 6 7
J J T

We can’t really generate any inferences off of this. We’ll have to move right to the answers. We’re
looking for something that “could be true:”

(A) This seems like a lot of work to figure out. Let’s defer judgment for now in hopes of finding more
obvious right or wrong answers down the line.

We know right away that we can eliminate (B). If we have J in week 4, we’d need G in week 3 (G before
every J), but we already have M in week 3! Eliminate (B).

We can also eliminate (C). Putting an M in week 4 means we have consecutive M’s in 3 and 4. This is
illegal. Eliminate (C).

We can eliminate (D), too! We can’t have T and J in 4 and 5. Remember, we must have a G before every
J. Get rid of it.

Wow. This is working out well. We can get rid of (E) as well. If we put T and M in 4 and 5, we’d get
MTM in 3, 4, and 5. But this doesn’t leave any room for a G between the two M’s!

This leaves us with (A). We could have taken the time to try (A) from the start, but it was much easier to
disprove the other choices. At this point, we would NOT want to spend the time to prove that (A) works,
but for the sake of exercise, let’s take a look:

Answer (A) puts G and T in 4 and 5. We’d have: _ _ M G T _ T. We still need a J, and before the J we’ll
need a G. There’s only one place for this: G J M G T _ T. We have to put our second M in, so an M will
go in the only remaining slot: G J M G T M T. This works! We have our M – G – M arrangement
squared away, and no other rules have been broken.

exactly 2M
G
T
M G M J
1 2 3 4 5 6 7 M
J GJ T
J J T
XX
17. (D)
Question Type: Unconditional
We don’t get any extra information here, so we’ll have to move to the answers. We’re looking for an
answer that “must be true.”

(A) Does G have to go in week 1 or week 2? This seems like a lot of work to figure out! Let’s defer
judgment on this one for now.

(B) Does M have to go in week 2 or week 3? Same goes for this one. We’ll defer judgment for now, in
hopes of finding a more obvious answer down the line.

(C) Do we have at most two G’s? There doesn’t seem to be anything preventing us from having three G’s.
We could have three G’s, two M’s, one J, and one T. It seems reasonable, but let’s defer for now.

(D) Do we have at most two J’s? Well, could we have three J’s? If we have three J’s, that means we have
to have three G’s as well (G before every J). So three J’s and three G’s would take up six of the seven
slots! What about our two M’s and our T in slot 7? We wouldn’t be able to fit all of this. So, this must be
true. We have at most two J’s. (D) is the answer.

GAME #4: Open Assignment

Conceptualize the Game


This is very similar to the second game. We have three centers (Center 1, Center 2, and Center 3) that
each recycle at least two and no more than three of five different types of materials (G, N, P, T, and W).
The exact number of materials recycled by each center is unknown. We can represent this as follows:

GNPTW

1 2 3

Will all the slots be filled? Not necessarily. Each center has the potential to recycle a maximum of three
materials. The boxes represent slots that must be filled (each center recycles at least two materials). Will
all the letters be used? Yes. The scenario says that “exactly five kinds of materials” are recycled at these
three centers. Will some letters be repeated? Well, we have at least six slots that must be filled and only
five letters, so we know for sure that there will be at least one repeated letter, if not more.

Setup
Let’s have a look at the constraints. The first constraint says that any center recycling W also recycles N.
We saw a constraint like this one in the last game! If we have W, we must also have N:
W

GNPTW
N

1 2 3

Notice that we have chosen to represent this constraint vertically to match the vertical columns of our
diagram. Pairing two letters together in the same recycling center means putting them in the same vertical
column. Thus, we want our notation to reflect this. The second constraint tells us that anything in Center 2
must also be present in Center 1. We’ll make a note on the diagram:

1 2

GNPTW
N

The last constraint tells us that exactly one 1center recycles 3 that center does not recycle G.
2 P, and that
From this constraint, we can infer that center 2 must NOT recycle P! Remember, anything center 2
recycles center 1 must recycle as well. If we put P with center 2, we’d also have to put it with center 1.

1 2

one P W
P GNPTW
G N

1 2 3
P

Again, notice that we’ve chosen to represent “no P and G together” vertically rather than horizontally.
The Questions

18. (B)
Question Type: Unconditional (Orientation)
This is another orientation question, so we can tackle this by taking one constraint at a time and
eliminating any answer choice that violates that constraint.

First constraint: If we have W we must also have N. (Remember, this does NOT work in the opposite
direction. In other words, if we start with N, this says nothing about having W.) In answer (A), Center 3
has W but no N. Eliminate (A).

Second constraint: Everything in center 2 must also be in center 1. This doesn’t mean that center 1 has to
be identical to center 2, but center 1 must contain anything that center 2 has. In answer (C), center 2 has T
but center 1 does not. Eliminate (C).

Third constraint: Only one P, and where there is P there is no G. Center 1 in answer (D) has P and G.
Eliminate (D). There are two P’s in answer (E). Eliminate (E). Answer (B) is the last one standing.

1 2

one P W

P GNPTW
G N

1 2 3

19. (D) P
Question Type: Unconditional
We don’t get any extra “If” statement here. We’re looking for a complete and accurate list of centers that
could recycle P. We already know that center 2 can’t recycle P, so let’s eliminate any answer that includes
center 2. Eliminate (C) and (E).

Now we just need to consider center 1 and center 3. Nothing yet tells us that they can’t have P, but we
want to be sure we haven’t missed anything. Notice that our correct answer (B) to #18 represents one
valid arrangement of the materials. In this arrangement, center 3 recycles P! If we’re confident in our
answer to #18 (which we should be), we don’t even need to test it. Knowing that center 3 can take P, we
can eliminate answer (A).

Down to (B) and (D). Now, what about center 1? Let’s do a quick drawing to test it out. We’ll put P in
column 1 and see if we can form a valid arrangement. We could put W and N in center 2 (which means
we’d have to put W and N in center 1 along with P), and T and G in center 3:
1 2
W

N W G

P N T

1 2 3
P

Let’s be sure it works. Everything in 2 is also in 1. P and G aren’t together. Wherever there’s a W there’s
also an N. We have exactly one P. Check! So center 1 can indeed have P. The correct answer is (D)

1 2

one P W
P GNPTW
G N

1 2 3
P

20. (C)
Question Type: Conditional
This question gives us a conditional piece of information: center 2 recycles three kinds of materials. Let’s
follow the inference chain as far as we can take it. If center 2 recycles three kinds of material (remember,
none of these can be P), then center 1 recycles the same three materials (center 1 must have everything
that center 2 has). Since all five materials must be recycled at least once, P must go somewhere! P must
be assigned to 3. Answer (C) is correct.

21. (D)
Question Type: Conditional
Here we get another piece of conditional “If” information: each center recycles three materials. Again, we
want to follow the inference chain as far as it will take us.
First inference: As in #20, if center 2 recycles three kinds of material (remember, none of these can be P),
then center 1 recycles the same three materials (center 1 must have everything that center 2 has). So, we
can eliminate (A), (C), and (E).

Second inference: Since all five materials must be recycled at least once, P must go somewhere! P must
be assigned to 3.

Third inference: If P is assigned to center 3, then G cannot be assigned to 3. Thus, both 2 and 1 will get a
G. Let’s get this down into an abbreviated diagram:

G G P

1 2 3

The only letters left to assign are W, N, and T. We’re forced at this point to consider answers (B) and (D).

(B) If only Center 3 recycles newsprint, then 1 and 2 can’t have N or W (since W triggers N). That only
leaves T for 1 and 2. But each center must have three materials! This answer doesn’t work.

Answer (D) is the last answer standing. We could verify it, but that wouldn’t be a great use of time.
1 2

one P W

P GNPTW
G N

1 2 3
P

22. (B)
Question Type: Conditional
The conditional “If” information in this case is that center 3 recycles G. Let’s see what inferences we can
derive from this. We know that P and G can’t be together, so center 3 can’t have P. We already know that
center 2 can’t have P, so center 1 must get P. Well, if center 1 gets P, then center 2 can only have two
materials (center 1 must have everything that center 2 has, and center 1 only has two spots remaining).
This also means that center 1 will have three materials (since 1 will get the two materials that 2 has). So
far we have:
P G

1 2 3

Since center 1 has P, it can’t have G, so 2 can’t have G either. So the options left for 2 are W, N, and T.
Eliminate (A).

Furthermore, 2 can’t have W without N, so it’s either WN or NT. Either way, center 2 gets N. (B) is the
correct answer.

1 2

one P W

P GNPTW
G N

1 2 3
P

23. (A)
Question Type: Conditional
The conditional information in this question is that center 1 is the only center to get W. Let’s follow the
chain of inferences again!

If 1 is the only center to get W, this means that 2 does not get W. This means that 2 gets exactly two
materials, and that 1’s other two materials will match 2’s. Since 1 must also get N (wherever there’s a W
there must also be an N), we get:
N

W N

1 2 3

Center 2 can’t get P, so center 1 can’t get it either in this case. Thus, P goes to 3. If P is in 3,
center 3 can’t get G. Thus, center 2 and center 1 get G:

N G

W N P

1 2 3

Thus, T must go to center 3. We end up with:

N G T

W N P

1 2 3

So, 1 and 2 are completely defined. None of the answers match 1 and 2, so the correct answer
must match 3. The only one that could work is (A).
Section II: Logical Reasoning

1. (B)
Question Type: Analyze Argument Structure
As is typical of many questions that ask you to identify the main conclusion, this argument starts
off with opposing points (points that run counter to the main conclusion), and then pivots on the
word “but,” which introduces the main conclusion. Answer (B) expresses the main conclusion
using slightly different words. The last sentence of the argument provides support for this main
conclusion (it is a supporting premise).

(A) is an overgeneralization that goes beyond the conclusion made in the argument.
(C) is out of scope. Employee ownership is not discussed in the argument, and thus cannot be the
conclusion drawn by the author.
(D) is true according to the passage, but this information is given as background information, not
the main conclusion.
(E) is also true according to the passage, but this statement is used as a supporting premise, not
as the main conclusion.

2. (B)
Question Type: Application
This argument is flawed in that it assumes that a mixture of elements (dogs) with two extreme
characteristics (lots of barking and very little barking) will produce an element with moderate
characteristics (moderate barking). Of course, this isn’t necessarily the case for any given
mixture. The argument in answer choice (B) contains the same flawed logic even if the elements
discussed (chemicals) are different.

(A) Jane is moderate in her studying, but she is not the result of a mixture.
(C) fails to make an argument about moderation.
(D) fails to make an argument about moderation.
(E) fails to make an argument about moderation.

3. (D)
Question Type: Inference
This argument makes a parallel between how people behave at the end of a century and how
people behave at the end of a life. If people look back at the events of their lives as their lives
come to an end, then we can infer that people will look back at the events of the century as the
century comes to an end. Answer choice (D) clearly expresses this.

(A) is incorrect because a century’s end will prompt reminiscence of the century’s events, not
reminiscence of the people’s lives.
(B) is out of scope. There is no indication that people would be fearful.
(C) looks forward instead of backward.
(E) is out of scope. There is no reason to conclude that people would look back on the
unfortunate events.
4. (A)
Question Type: Identify the Flaw
Danto Foods has a competitive motive to claim that Ocksenfrey’s products are devoid of
nutritional value, but that doesn’t necessarily mean that Danto Foods is lying. Take the following
analogous argument:

John and Sally are competing in a spelling bee. John said that Sally spelled her last word
incorrectly. But John really wants to win, so this means that John’s claims are false and that
Sally actually spelled her word correctly.

Isn’t it possible that John wants to win AND that Sally spelled her word incorrectly? Just because
John is biased doesn’t necessarily mean that his claims are false. Answer choice (A) clearly
expresses this type of flaw.

(B) is incorrect. We have no information on the sample size.


(C) is out of scope. Ocksenfrey’s motivations are irrelevant. We care only about Danto’s
motivations, since these motivations provide the support for the final conclusion.
(D) is out of scope. We don’t care about Danto’s meals. The conclusion of the argument focuses
only on Ocksenfrey’s meals.
(E) is out of scope. Danto Foods’ products are not at issue here.

5. (B)
Question Type: Weaken the Conclusion
The scientist concludes that the buildup of minor gases in the atmosphere is the primary cause of
the earth’s warming. To weaken this argument, we want to look for a counter premise that either
(1) casts doubt on the minor gas theory, or (2) presents a more compelling alternative cause for
the warming. Answer (B) certainly casts doubt on the minor gas theory. If most of the warming
occurred before the minor gas buildup, it doesn’t make much sense to say that minor gases were
the cause.

(A) is out of scope. The source of the gases is irrelevant.


(C) is very tempting, as it provides what seems to be an alternative explanation: more solar
radiation would mean higher temperatures. But, this does NOT say that the solar radiation was
higher overall in the last century than in previous centuries, but simply that some years had
higher solar radiation than others during the last century.
(D) is irrelevant. We care about what’s keeping heat in not what’s keeping heat out.
(E) can be an attractive answer, but be careful. Answer (E) actually strengthens the argument!
This is the exact opposite of what we want.

6. (B)
Question Type: Assumption
In order to be appointed to the executive board, one must have an undergraduate degree and be
free of felony convictions. Murray has an undergraduate degree, but he has a felony conviction,
so he cannot be appointed to the executive board. In drawing his/her conclusion the author
assumes, however, that the qualifications for the executive board are the same as the
qualifications for the position of Executive Administrator. Did you catch this shift in detail?
Answer (B) correctly expresses the assumption made by the author. Notice that the assumption,
when stated explicitly, actually fixes the hole in the argument.

(A) is not a required assumption.


(C) is not a required assumption. In fact, the argument seems to assume that an undergraduate
degree is required for the position.
(D) is tempting, but it fails to close the gap between executive board and Executive
Administrator.
(E) is not a required assumption. It doesn’t matter if the felony charge is relevant to the job or
not.

7. (D)
Question Type: Application
Answer (D) is the only answer that presents a person acting free of calculated self-interest or the
desire to adhere to societal norms. In (D), Jadine’s actions are based on an abstract principle that
protecting the environment is more important than monetary gain.

(A) describes Bobby as one who is influenced by how others perceive him.
(B) describes Wes as one who is influenced by how others perceive him.
(C) describes Donna as one who is influenced by how others perceive her.
(E) describes Leigh as one who is influenced by how others perceive her.

8. (A)
Question Type: Inference
Proponents of the electric car are expecting an “abatement” of environmental degradation, but
they aren’t considering the effects of charging the batteries. Thus, the consequences of
introducing electric cars will likely be worse than they expect. Answer (A) expresses this.

(B) is incorrect because we know nothing of the relative popularity of electric versus other cars.
(C) is incorrect. Certainly technical problems must be solved (battery), but are we sure there
aren’t other problems as well? Be careful of the word “purely.”
(D) is incorrect. We know there will be emissions as a result of charging the car batteries, but do
we know that the total emissions will increase? Not necessarily.
(E) is incorrect. We know that charging batteries with nuclear or coal-fired plants will cause
emissions, but we don’t know if the level of emissions will be higher or lower than previous
levels.

9. (E)
Question Type: Weaken the Conclusion
What are the potential problems with the argument? In this case, the author assumes that no other
part of the population will take the place of the 13 to 16 year olds when it comes to purchasing
games. If we can find an answer that destroys this assumption, we’ll have our answer. Answer
(E) does just that.
(A) actually strengthens the argument, the exact opposite of what we want.
(B) is out of scope (video game rentals?).
(C) is out of scope (new entertainment options?).
(D) is out of scope (different types of video games?).

10. (B)
Question Type: Analyze Argument Structure
This argument starts with the main conclusion, and answer (B) does the best job of expressing
this conclusion (in slightly different words). Everything else in the passage is used to support this
main conclusion.

(A) is attractive because it’s true according to the text. However, (A) restates a supporting
premise of the argument, not the main conclusion of the argument. The fact that they are biased
in their interpretations is reason to use double-blind techniques.
(C) may be reasonable as a conclusion that a normal person might draw, but it is not the
conclusion stated in the passage.
(D) is attractive because it restates part of the text. Furthermore, this feels like a claim
(conclusion), but is it the final conclusion? No. It is an intermediate conclusion that supports the
final conclusion: Scientists should avoid misinterpretations, THEREFORE they should use
double-blind experiments whenever possible.
(E) may be implied by the passage, but it is not the conclusion. The conclusion is that they
should be used when possible.

11. (C)
Question Type: Analyze Argument Structure
This is very tough. This question asks us to determine the function of a particular part of the
argument. It’s important to determine whether the portion in question supports or opposes the
author’s main conclusion. In this case, it is used as support for the conclusion, since it is an
example of a previous and similar change in (not devolution of) the human mind. Answer (C)
clearly expresses this role.

(A) describes the part in question as supporting the point made by the common complainers
instead of supporting the author’s claim.
(B) misstates the conclusion (general hypothesis).
(D) is very attractive, but it’s not quite right. Let’s look at an analogous argument to show why
not (the italicized part is the part for which we need to determine the function):

My fitness trainer argues that weight lifting will destroy my quickness. But last year he
complained that working on my quickness would destroy my endurance. So, most likely, weight
lifting will change my overall fitness, but will not destroy it.
The trainer thinks that weight lifting will destroy the author’s quickness. Notice that the author
does NOT dispute this! Rather, the author concludes that weight lifting will change her overall
fitness (maybe even conceding that her quickness will be destroyed). The trainer and the author
are not arguing the same central point! The same holds true in #11. The common complaint is
that electronic media will destroy literary skills. The author argues that electronic media will
alter but not destroy the human mind in general. Answer (D) is wrong because it suggests that
the author is arguing in direct opposition of the common complaint. Not true. In fact, the author
may even agree that literary skills will be destroyed as the human mind is altered.
(E) describes the part in question as supporting the point made by the common complainers
instead of supporting the author’s claim.

12. (A)
Question Type: Application
The original argument begins by setting up a mutually exclusive relationship between two things.
In other words, it describes two goals (keeping a confidence and answering truthfully) that
cannot both be achieved simultaneously. It uses this mutual exclusivity to conclude that one
cannot expect to achieve both goals. We want to find an answer that sets up the same logical
structure: introduction of a mutually exclusive relationship, conclusion that the two parts of the
relationship cannot be achieved simultaneously. Answer (A) is the only one of the five choices
that does this. One way to differentiate the choices is to compare the conclusions for each: which
one concludes that two parts cannot be achieved simultaneously?

(B) fails to explicitly set up two mutually exclusive parts.


(C) fails to conclude that two parts cannot be achieved simultaneously.
(D) fails to set up two mutually exclusive parts.
(E) is tempting. It concludes that one path must be taken because the other path is impossible to
take. However, it does not explicitly conclude that the two paths cannot be achieved
simultaneously.

13. (C)
Question Type: Assumption
What is the author assuming in making the conclusion that M contains twice as many cans as L?
Well, suppose that L contained 10 units of aluminum, but that only 7 units were recovered in the
recycling process. All 7 units were used to create M. According to the passage, 7 represents half
of M’s aluminum, so M had 14 units in total. Can we conclude that L originally had half as many
cans as M? No! 10 is more than half of 14. In order for this argument to work, the author must
assume that no aluminum is lost in the recycling process. Answer (C) correctly expresses this
assumption.

(A) is out of scope. This argument focuses on taking L’s cans and creating M’s cans. We don’t
care what happens after.
(B) is out of scope. The quality of the aluminum is irrelevant
(D) is out of scope. This argument focuses on taking L’s cans and creating M’s cans. We don’t
care what happens before.
(E) is out of scope. Can made from other materials are irrelevant.
14. (E)
Question Type: Weaken the Conclusion
The author concludes that microwaves, not heat itself, destroy substance X. To weaken this
argument, we’d want to show that in fact heat could play a role, or, alternatively, we’d want to
show that microwaves do not play a role aside from providing heat. The only answer that is
relevant to the conclusion is answer choice (E). If microwaves create small heat pockets in the
liquid that rise to temperatures much higher than the temperature of the overall liquid, then it’s
possible that it’s actually these high heat pockets that destroy the substance instead of the
microwaves. This would explain why the 50 degree heat from a conventional source destroyed
almost none of the substance but that the same 50 degree overall heat in a microwave destroyed
half of the substance.

(A) is irrelevant.
(B) is out of scope. It doesn’t matter if the destroyed enzymes can be replaced.
(C) is out of scope. It compares the rate at which the liquid is heated between two conventional
sources of heat. We’re interested in the difference between a conventional source and
microwaves.
(D) is out of scope (taste?).

15. (D)
Question Type: Assumption
Each year’s vaccination will protect only against the strain of the virus deemed most likely to be
prevalent that year, so high-risk individuals will need to receive a vaccine for a different strain
each year. Doesn’t this assume that the prevalent strain of the virus will change every year?
Take, for example, a hypothetical scenario in which last year’s prevalent flu strain was the same
as this year’s. High-risk individuals would thus be getting the same vaccination this year that
they got last year. In order to conclude that people will get vaccinated for a different strain each
year, the author must assume that the flu strain will change each year. Answer (D) correctly
expresses this assumption.

(A) is out of scope. The number of vaccinations given is not at issue.


(B) is out of scope. The likelihood of an outbreak is irrelevant.
(C) is tempting, but notice the language in the passage: the annual vaccination will protect
against just one strain – the one deemed most likely to be prevalent that year. Thus, the fact that
no vaccine protects against multiple strains is a moot point.
(E) is out of scope (side effects?).

16. (D)
Question Type: Analyze Argument Structure
In this case, Taylor concludes that the researchers’ claim is suspect. Notice that Sandra never
addresses the researchers’ claim. In fact, Sandra never addresses the research (verbal vs.
nonverbal signals) mentioned by Taylor at all! Since neither the research nor the researchers’
claim is a point of overlap between the two participants, they can’t possibly disagree over these
things. However, they do disagree over the supporting premise used by Taylor. He says: “…
claims of such exactitude could never be established by science.” Sandra addresses this head-on:
“Many scientific disciplines obtain extremely precise results.” Answer (D) correctly highlights
this area of disagreement.

(A) is a half-scope answer. Sandra never addresses the research on nonverbal communication.
(B) is a half-scope answer. Sandra never addresses the research on nonverbal communication.
(C) is a half-scope answer. Sandra never addresses the research on nonverbal communication.
(E) is incorrect. Taylor states that scientific claims are suspect, not necessarily that they are false.

17. (B)
Question Type: Identify the Flaw
This argument is flawed because of the use of evidence from a limited perspective. The hospital
executive uses the testimony of computer experts (with a limited set of expertise) in order to
make a conclusion about what the hospital’s main priority should be (a very broad strategic
decision). Answer (B) identifies this flaw.

(A) is incorrect. A problem is introduced, but the cause of that problem is not discussed.
(C) is incorrect. Confusing correlation with causation is indeed a common flaw, but it is not the
issue with this argument.
(D) is tempting (unrepresentative sample, or inadequate sample size, is often used as a flaw on
the LSAT), but in this case we know nothing about how the computer experts came to their
conclusion about the biggest threat facing large institutions. The nature of the sample is
unknown.
(E) just isn’t true. This inference is not made.

18. (B)
Question Type: Inference
Our job is to find the answer that is most easily proved by the information in the passage.
Answer (B) is nice and safe. Given the first two sentences of the passage, it’s hard to argue
against (B). While it may not be completely provable, it’s certainly the best of the bunch.

(A) is in direct contradiction with the passage. In fact, this reluctance is described as the
foundation of modern science.
(C) goes far beyond the text in stating that there is conclusive evidence. We have no support for
that in the passage.
(D) is incorrect. We actually don’t know if skeptical scientists have offered alternative
hypotheses.
(E) goes a bit too far in stating that recognition is the “primary” motivation.

19. (A)
Question Type: Strengthen the Conclusion
The historian concludes that the Land Party was successful in part because it focused its efforts
in 1935 on rural and semi-rural areas, areas hit hardest by the economic troubles. Notice that
answer choice (A) discusses the Land Party’s lack of effort to address the interests of urban
groups in earlier elections. This is irrelevant to the argument, and is therefore the correct answer
(since it does not strengthen the argument).

(B) strengthens the argument.


(C) strengthens the argument.
(D) strengthens the argument
(E) strengthens the argument.

20. (E)
Question Type: Analyze Argument Structure
In stating that the 15 opposing votes represent far less than 1 percent of Hopeville’s population,”
Gamba is in essence saying that the sample size of the vote is too small to take seriously. Answer
(E) expresses this.

(A) is tempting because it addresses the data associated with the vote. However, Gamba never
claims that the vote was inaccurate because the opposition was “more likely to vote.”
(B) is tempting as well because it addresses the data associated with the vote. However, Gamba
never claims that the data has been manipulated, only that it is not representative.
(C) is incorrect. Munoz never claims that the result of the vote guarantees citywide opposition.
Thus, it is impossible for Gamba to refute this guarantee.
(D) is confusing in its language. Gamba doesn’t refute Munoz’s argument on the grounds that the
vote is impossible to disconfirm, but rather on the grounds that the vote isn’t representative of the
larger population.

21. (A)
Question Type: Identify the Flaw
This is a classic “cause vs. correlation” issue. Ownership of a minivan is correlated with lower
accident rates, but in no way does this mean that owning a minivan causes one have a lower risk
of accident (maybe those who own minivans are just naturally more cautious people in the first
place). Thus, the driver is incorrect in concluding so. Answer (A) is correct. An equivalently
flawed argument would be:

Many rich people eat caviar. Therefore, if I begin eating caviar I will get rich.

This is, of course, a ridiculous argument because it confuses a correlation with a cause. Eating
caviar does not cause one to be rich.

(B) tries to tempt us with the common “sample size” flaw language, but we know nothing about
the sample size used in the driver’s research.
(C) is incorrect. This isn’t a case of likely versus certain.
(D) tempts us with familiar conditional logic language: sufficient and necessary conditions.
Here’s an example of mistaking a sufficient condition for a necessary condition: “Switching
from a sports car to a minivan (sufficient condition) causes the driver to drive more cautiously
(necessary condition). Because John has begun driving more cautiously, he must have switched
from a sports car to a minivan.” No! Not necessarily true. The conclusion in this argument has
mistaken a sufficient condition for a necessary condition. This is another way of saying that the
argument has reversed the logic. #21 does not contain this type of flaw.
(E) is ambiguous at best. Which source are we talking about? Friends? Research? Either way, we
have no way of knowing if they are well-informed or not.

22. (D)
Question Type: Inference
This argument is presented in a linear fashion:

lack of thorough news coverage + secret political process  isolate politicians from electorate
 small chance of official response to resident participation  discourages participation

Answer (D) is a very safe choice. There are two initial sources for the issue (see diagram above).
More frequent media coverage “would reduce at least one source of discouragement.”

(A) is very tempting, but it’s tough to prove. The fact that politicians are isolated is one reason
for the small chance of official response, but making them less isolated doesn’t account for the
other reason: lack of thorough news coverage. So if we fix one problem, does that guarantee that
an official response will now be “likely?” Hardly. This would be like saying: “Janice can’t run
the marathon because she is not in shape and she doesn’t own running shoes. If we get her some
running shoes, she’ll be able to run the marathon.” What about the fact that she isn’t in shape?
(B) makes an irrelevant value judgment.
(C) goes too far in saying “the most significant factor.”
(E) sets up an illogical causal relationship.

23. (C)
Question Type: Assumption
Before we even get to the answer choices, we can spot a gap in this argument. An action is
wrong if, and only if, it reduces the aggregate well-being of the people affected by it. The author
concludes that any action that does NOT reduce the aggregate well-being is by default right. The
author assumes that there are only two kinds of actions: right and wrong. Answer choice (C)
expresses this assumption in clear terms.

(A) is incorrect. We’re interested in actions that leave the aggregate well-being unchanged. What
do we call these actions?
(B) is tempting, but it misses the issue. We don’t care if an action can be both right and wrong.
The issue is whether it can be neither.
(D) misses the issue as well. We don’t care if there are such actions, but rather what to call them
(right, wrong, or something else) if they did exist.
(E) is out scope. “Consequences” in general is a concept that is beyond the scope of the
argument.

24. (A)
Question Type: Inference
This is an inference question in disguise. “Proposition” is codeword for conclusion, so this
question is asking us to draw a conclusion, or inference, from the information in the passage. On
inference questions, we want the answer that is easiest to prove given the information in the
passage. Answer (A) uses soft language that is difficult to argue with: can contribute to
successful product design.

(B) is incorrect, as we don’t know if companies “traditionally” conduct these surveys, nor do we
know if these surveys are “extensive.”
(C) is wrong as we have no evidence to indicate that companies aim for specific “market niches.”
(D) says “a car will have unappealing features.” This is tough to justify.
(E) places external features in opposition to internal features. The passage does not explicitly
distinguish between the two.

25. (C)
Question Type: Explain a Result
The fact that there was innovation in painting but not sculpture is surprising considering that
both forms of art were being sponsored by the French Academy (which frowned on innovation).
Why innovation in painting and not sculpture? We want to choose an answer that explains this
apparent discrepancy. (C) clarifies the result. Many of the innovative paintings were coming
from painters who were not sponsored by the Academy, and therefore were not bound by the
Academy’s restrictions.

(A) makes the unexpected result even more unexpected. If the academy gave more support to
painting, we’d expect painting to be more restricted by the Academy’s preferences (no
innovation). But, we actually got the exact opposite: more innovation in painting.
(B) is similar (it makes the result even more unexpected). More money to painters would lead us
to assume that painters would be more restricted.
(D) is irrelevant. The number of artists who were both painters and sculptors doesn’t matter.
(E) is irrelevant. The key is the difference in funding between painters and sculptors, not the
overall trend of funding in general.
Section III: Logical Reasoning

1. (C)
Question Type: Application
People sometimes sacrifice sensual comfort or pleasure for the sake of appearances (impressing
others). We want to choose an example that illustrates this principle. In (C), the couple sacrifices
their favorite wine in order to impress their guests. This is a perfect illustration of the principle.

Each of the wrongs choices is either missing the “sacrifice” part or the “for the sake of
appearances” part of the principle.
(A) is incorrect because there is no indication that the person has purchased the car for
appearances.
(B) is incorrect because neither the child nor the parent is sacrificing sensual comfort.
(D) is a bit closer. The person might be sacrificing sensual comfort, but not for appearances.
(E) is just the opposite. The acrobat may be trying to impress others, but he/she isn’t sacrificing
sensual comfort.

2. (A)
Question Type: Explain a Result
If Jimmy got a new, highly efficient water heater, how could his gas bills go up? We’d expect the
bills to go down. We can try to anticipate some possible explanations. Most likely, Jimmy’s gas
usage increased after he bought the new water heater. This is a tricky EXCEPT question. Four of
the answer choices will help to explain the unexpected result while one will not. In fact, the
answer we want to choose will either be irrelevant to the situation at hand or it will intensify the
unexpected result. Answer (A) is the only answer that does NOT offer an explanation. The
relative percentage doesn’t give us any information, as percentage figures don’t tell us anything
about the actual amount of gas used.

(B) helps to explain the result. If the size of the household doubled, then gas usage would
probably double, which would explain the higher bills.
(C) helps to explain the result. A new appliance that uses gas would explain the higher gas bills.
(D) also helps to explain the higher bills. Higher rates mean higher bills.
(E) contributes to an explanation as well. Increased usage would mean higher bills.

3. (E)
Question Type: Analyze Argument Structure
Carolyn argues that the work is NOT a portrait because it does not resemble the subject. Arnold
disagrees, but with what? Does he disagree that it’s not a portrait or that it doesn’t resemble the
subject? Arnold’s second sentence gives us the answer. He states that the work is a realistic
portrait. Notice that Arnold never takes direct issue with Carolyn’s claim that a portrait must
resemble the subject. Answer (E) gets at the heart of the disagreement: whether the work is a
portrait.

(A) is out of scope because neither person questions whether the work should or should not be
considered art.
(B) is out of scope because neither person questions whether the work is actually Quinn’s.
(C) is a half-scope answer. Carolyn discusses whether the work bears a resemblance to the
subject, but Arnold does not.
(D) is a half-scope answer. Arnold discusses the fact that the work “contains instructions,” but
Carolyn does not.

4. (E)
Question Type: Identify the Flaw
The author concludes that the posters are unlikely to boost employees’ motivation. The reasoning
used to make this conclusion is that these employees are already motivated. If we read critically,
we can anticipate the flaw in this argument. The argument assumes that someone who is already
motivated can’t become even more motivated. Answer (E) clearly expresses this flaw.

(A) is incorrect because this is exactly what the argument is considering.


(B) is wrong because it doesn’t matter at all if the corporations in question (“these corporations”
as stated in the conclusion of the argument) are representative of corporations in general. The
argument is concerned only with the corporations mentioned.
(C) may be true, but it is outside the scope of the argument. The argument pertains to the posters’
effect on motivation. Any other possible effect is irrelevant.
(D) is incorrect for the same reason. Other factors that might contribute to productivity are
outside the scope of the argument. We care only about the posters’ impact on motivation.

5. (C)
Question Type: Assumption
What if the items being dumped were actually food items? Couldn’t the ants have been emptying
the dumping site and taking food particles to their neighbors simultaneously? Atrens assumes
that the dumped items were not food. Answer (C) clearly expresses this assumption.

(A) is out of scope (human interactions?).


(B) is out of scope. Whether or not the ants were giving food gifts is in question, but their
capacity to receive and make use of gifts is irrelevant.
(D) is wrong because the reception of the food gifts is irrelevant and out of scope.
(E) is irrelevant as well. It doesn’t matter if the entomologist changed his mind. What’s at play in
this argument is whether he/she was right or wrong the first time.

6. (B)
Question Type: Application
The passage is an example of a good deed actually working out well for the person doing the
good deed. Answer (B) expresses this.

(A) misses the point, and also uses language that is too extreme: the “only” way?
(C) is certainly not supported by the passage.
(D) is very tempting, but can we say that it is “usually” in one’s “best” interest to perform good
deeds? This language goes a bit too far. Compare it with the correct answer (B): good deeds
“sometimes” have positive consequences.
(E) is too extreme: “must” have community support?

7. (B)
Question Type: Analyze Argument Structure
This is a very tricky conversation. Antonio states that one can live a life of moderation by staying
on the middle course. Maria disagrees. She says that one who stays on the middle course and
never takes risks is actually not living a life of moderation (because never taking risks is not, by
definition, moderate). Thus, the two disagree over what it means to live a life of moderation.
Answer (B) expresses this. Notice that Maria never addresses the rest of Antonio’s argument
about all the things that moderate people miss out on.

(A) is wrong because Maria never discusses whether it is desirable or undesirable to take
chances. She only comments on what it means to live a life of moderation.
(C) is incorrect, because other virtues are out of scope.
(D) is incorrect. How often a person ought to deviate from the middle course is out of scope.
Only Antonio discusses whether it is desirable to be spontaneous.
(E) is a half-scope answer. Only Antonio discusses whether it is desirable to be spontaneous.

8. (E)
Question Type: Identify the Flaw
We can anticipate the flaw in this argument even before reading the answer choices. The test
allowed consumers to compare a towel washed with Fabric-Soft with a towel washed without it.
Thus, they could conclude that they like the Fabric-Soft towel more, but they cannot necessarily
conclude that Fabric-Soft is the most effective fabric softener. After all, they haven’t even tested
any towels washed with other softeners. The advertisement fails to consider whether the
consumers tested other softeners. Answer (E) expresses this.

(A) is incorrect. Whether or not consumers are allergic is irrelevant


(B) is irrelevant. The environment is not at issue in this argument. The focus is whether Fabric-
Soft is the most effective softener.
(C) is incorrect because cost is out of scope.
(D) is wrong as well. Cost-benefit is not the issue. The only thing at issue is whether Fabric-Soft
is the most effective at doing its intended job (regardless of how much it costs).

9. (D)
Question Type: Assumption
This argument is tricky. The conclusion made by the naturalist is that recent claims that the tiger
is not extinct are false. In other words, his/her conclusion is that the tiger is indeed extinct. The
evidence presented is that the tiger’s natural habitat was taken over by sheep farming, and that
naturalists have no evidence of the tiger’s existence in the region. But, isn’t the author assuming
that the tiger hasn’t moved to another region? Answer (D) expresses this assumption. Notice that
when the assumption is explicitly stated, it helps to support the argument (“We can’t find
evidence of the tiger here, and they haven’t moved somewhere else, so they no longer exist.”),
and that this assumption is absolutely necessary in order for the conclusion to be drawn.
(A) is tempting because it would support the argument. However, the author doesn’t need to
assume that the tiger became extinct because of starvation.
(B) actually weakens the argument. If scavengers can destroy tiger carcasses, this may explain
why naturalists haven’t been able to find evidence of the tiger’s existence (a reason other than
extinction). This can’t be a necessary assumption if it goes against the argument.
(C) is tempting, but the author doesn’t necessarily need to assume that every naturalist has
looked systematically for the tiger.
(E) is not a necessary assumption either. The author already states that the sightings are alleged
sightings, so it is not necessary to assume that those who have reported sightings are
inexperienced.

10. (E)
Question Type: Inference
If a person can be encouraged to like something by linking that thing with a picture of something
else that the person already likes, then wouldn’t advertisers want to put pictures of things we
already like next to the product they’re trying to sell us? A perfect example: cereal boxes with
pictures of Disney characters on the back. Advertisers know that kids like Disney, so they put
pictures of the characters on the back in hopes that kids will then like their cereal as well.
Answer (E) expresses this concept. Answer (E) is safe and fairly easy to prove given the
information in the passage.

(A) is tempting, but the passage states that the method is effective if pictures are used in addition
to prose, not instead of prose.
(B) confuses the issue. Advertisers are trying to use pictures to encourage people to like their
products, not trying to encourage people to like products that can more easily be represented in
pictures (same words, very different meaning).
(C) is incorrect. We can’t really conclude that advertisers will prefer TV over magazines. After
all, they can use pictures in magazines just as easily as they can on TV.
(D) is wrong as well. We have no indication that advertisers would use pictures to negatively
represent competitors’ products.

11. (E)
Question Type: Assumption
The author is assuming that nothing else (aside from the amount of mercury in the eaten fish)
could account for the difference in the amount of mercury found in the feathers. More
specifically, the author assumes that the process of preserving the old birds didn’t somehow
decrease the mercury levels of the old birds’ feathers. Answer (E) expresses this assumption.
Notice that when explicitly stated the assumption supports the argument: Today’s bird feathers
have less mercury, and the process of preserving the old birds didn’t decrease the mercury levels
in old bird feathers, so the old fish must have had less mercury. The argument depends on this
italicized assumption.

(A) is very tempting because it seems to provide an alternate explanation for why today’s bird
feathers have more mercury (birds back then didn’t eat as much fish), but this would actually
weaken the author’s argument. Remember, we want an assumption that is necessary to the
author’s argument, not one that destroys the argument.
(B) is wrong. The issue is whether the fish today have more mercury, not why they have more
mercury.
(C) is irrelevant to the conclusion of the argument.
(D) is tempting, but it’s very similar to (A) in that it seems to provide an alternate explanation for
why today’s bird feathers have more mercury (the old feathers were taken from birds that
weren’t fully grown), but this would actually weaken the author’s argument.

12. (D)
Question Type: Analyze Argument Structure
The argument starts with background information and an opposing point (the novels are similar,
so plagiarism might be involved), but then pivots to the author’s main conclusion with the word
“however.” The author concludes that it’s probably not plagiarism, but rather coincidence. The
author then supports his/her conclusion with a supporting premise at the end of the argument.
This is a very common argument structure: OPPOSING POINT --- CONCLUSION ---
SUPPORTING PREMISE. Answer (D) correctly expresses the main conclusion.

(A) restates some of the background information given for the argument.
(B) restates the opposing point.
(C) restates the supporting premise.
(E) gives a cause/effect relationship that does not accurately represent the conclusion made in the
passage.

13. (B)
Question Type: Strengthen the Conclusion
One way to strengthen an argument is to provide a piece of information that closes a gap in the
argument. In other words, we can strengthen this argument by explicitly stating one of the
assumptions. Let’s rearrange this argument in order to make the logic more transparent.
Cognitive psychotherapy changes conscious beliefs. Other forms of psychotherapy change
unconscious beliefs. Since only conscious beliefs are under the direct control of the patient,
cognitive psychotherapy is likely to be more effective at treating psychological problems.

What’s the assumption? The author assumes that in order to be effective, a treatment must
address mental processes that are under the control of the patient. This is easy to miss, but it’s an
important assumption that is critical to the argument. We can strengthen the argument by making
this assumption explicit. Answer (B) does this.

(A) weakens the argument.


(C) is irrelevant. It doesn’t matter if it’s the only form of therapy that focuses on conscious
beliefs. The argument deals only with cognitive psychotherapy vs. other forms of psychotherapy
that do NOT focus on conscious beliefs.
(D) strengthens the argument by providing an additional piece of supporting evidence, but it does
not strengthen as much as answer (B). An answer that fills a gap in the argument will generally
strengthen the most (since the argument is weak with the gap).
(E) fails to provide any information comparing the two types of therapy.

14. (C)
Question Type: Application
The author concludes that universities should use open-source software because the values
embodied in open-source software match those of academia (open, transparent). This is a valid
conclusion to draw if one holds the belief that universities should generally choose software that
matches the values of the university. The argument is created on the basis of this principle.
Answer (C) states this principle.

(A) is incorrect. The argument holds that software should be chosen based on the values it
embodies, NOT on how advanced it is, or how much it might help a university meet its goals.
(B) is wrong for similar reasons. Cost has nothing to do with it.
(D) is very tempting, but it is incorrect because of the word “efficient.” Software that matches
the values of the university will not necessarily be the most efficient (at least not according to the
argument).
(E) is out of scope (any activity?).

15. (C)
Question Type: Weaken the Conclusion
This argument has many issues and many assumptions. For one, the argument is based on data
that may contain sample bias. A higher percentage of the “more than 6 months” group reports
being satisfied with the treatment. But these are people who have opted to continue treatments!
Isn’t it likely that the “more than 6 months” group is a self-selecting group of people who tend to
like the treatments, and so elect to continue with the treatments? Here’s an analogous case:

60% of those who have seen the Star Wars trilogy exactly one time report having liked the
trilogy. 95% of those who have seen the trilogy two times or more report having liked the
trilogy. Thus, the more one sees the trilogy, the more one likes the trilogy.

No! Not necessarily. Maybe the people who already liked it the first time opted to see it multiple
times. This would obviously skew the data. Answer (C) expresses the possibility of a biased
sample, which would certainly weaken the argument.

(A) is irrelevant to the argument (we already know that 36% said things got better, so it’s no
surprise that some of the others said things got worse).
(B) is very tempting, but keep in mind that the response rate is irrelevant. The argument is based
on those who have responded.
(D) actually strengthens the argument by raising the possibility that the number of “more than 6
months” patients who are satisfied is actually underrepresented by the survey.
(E) is irrelevant to the conclusion.
16. (B)
Question Type: Inference
A nation does not have moral rights and responsibilities. But in order to survive, citizens of that
nation must believe that the nation does have moral rights and responsibilities. Thus, in a
surviving nation, some of its citizens hold false beliefs. Answer (B) expresses the undeniable
truth of this statement and is therefore the correct answer.

(A) is not true. The nation can continue to exist if some if its citizens hold the false belief that the
nation has moral rights and responsibilities.
(C) is too extreme (never?). Never?
(D) certainly isn’t provable.
(E) is too extreme as well (always?). It’s true that thinking of a nation in metaphorical terms
(assigning rights and responsibilities) helps to encourage sacrifices for that nation, but to say that
nations should always be thought of in such a way is too much.

17. (B)
Question Type: Assumption
The argument seems reasonable, but consider someone who may, for whatever reason, have
unequal strength on either side of his spine to begin with. Wouldn’t he want to exercise the
weaker side more than the stronger side in order to make them equal? The assumption in the
argument is that unequal exercise leads to unequal muscle strength (in fact this may not be true
for all people). Answer (B) clearly expresses this assumption.

(A) is incorrect. The argument does not need to assume that equal muscle strength will be
enough on its own to keep the spine aligned (other things might be necessary as well). The
argument says that equal muscle strength is one thing that is required to keep the spine aligned,
but it doesn’t have to be enough on its own.
(C) is incorrect for the same reason. The argument doesn’t assume that equal exercise will
necessarily lead to a healthy back, but simply states that equal muscle strength is one thing that is
necessary for a healthy back.
(D) is way too extreme and certainly not an assumption required by the argument.
(E) is not a required assumption either. The author doesn’t necessarily assume that the exercise
will be daily exercise. Remember, the answer we choose must be a required assumption.

18. (B)
Question Type: Identify the Flaw
The author assumes that most people would place higher moral value on protecting a family
member than following laws. But what if this isn’t the case? What if most people actually think
that the morality of following laws has higher moral value than protecting a family member? The
author fails to consider this. If it were true, the argument would be destroyed. Answer (B) is
correct.

(A) is incorrect. The argument does NOT make a broad generalization. Notice the phrase
“sometimes morally right” in the passage. If we say “Kelly is 5 years old and she eats
chocolate,” we can also say “some children (at least one in this case) eat chocolate.” The word
“some” allows us to make this statement. “All” children would be too broad a generalization.
(C) is wrong. This presumption is never made in the argument.
(D) is incorrect. The argument never takes for granted that there is no obligation to follow laws.
(E) is not right either, because the passage clearly states that the person is “known” to be falsely
accused.

19. (A)
Question Type: Strengthen the Conclusion
What candidates promise before they are elected and what they actually do once they are elected
are sometimes two very different things. In order for the conclusion to hold, the author must
assume that campaign promises will be kept (the government will intrude by raising taxes). The
argument will be strengthened if this assumption is made explicit. Answer (A) does this.

(B) is the exact opposite! It weakens the argument.


(C) is out of scope (most common problems?).
(D) is out of scope (a comparison of democratic systems versus non-democratic systems?).
(E) is also out of scope. We don’t care what happens to politicians who promise to do what they
actually believe ought to be done. The passage clearly states that politicians who promise to help
voters are the ones who get elected. What matters is whether they keep these promises, NOT
whether they believe in these promises.

20. (C)
Question Type: Application
The main reason given for accepting the proposal is suspicion of the other side’s (the historical
society’s) motives in opposing the proposal. We want to choose an answer that uses the same
flimsy reasoning. Answer (C) does this. The reason given for suggesting that one should have no
more than one haircut per month is suspicion of the beauticians’ suggestion that one should have
two cuts per month.

(A) is tempting because it mentions parties that oppose the main conclusion, but it does not use
suspicion of the opposition’s motives in order to justify the conclusion that significant works of
art should be safeguarded.
(B) does not mention any opposition to the main conclusion.
(D) does not mention any opposition to the main conclusion (the residents are in favor of
postponing the construction).
(E) does not mention any opposition to the main conclusion.

21. (B)
Question Type: Weaken the Conclusion
The conclusion is that the consumption of meat is morally unacceptable because raising meat
takes away valuable grain resources that could feed 16 times as many people. But, this would be
a valid argument only if raising meat uses valuable grain resources. In other words, what if it
were possible to raise meat without using any valuable grain? Then, we could raise meat without
feeling guilty about doing it at the expense of grain, and the argument of immorality would be
severely weakened. Answer (B) expresses this idea.

(A) is out of scope (people’s preferences are irrelevant to a discussion about morality).
(C) is out of scope (the issue isn’t nutritional value of grain vs. meat, but rather whether we can
continue to raise meat).
(D) is the most tempting of the incorrect answer choices. It seems to weaken the argument by
suggesting we could save this prime farmland for grain production (making it more feasible to
raise both meat and grain, and thus morally acceptable to do both), but could we still raise meat
without feeling guilty about using up 16 pounds of grain per pound of meat raised? Not
necessarily.
(E) is out of scope (the issue isn’t whether a grain-only diet is adequate, but whether it is morally
acceptable to eat meat).

22. (C)
Question Type: Inference
Let’s follow the cause and effect chain given in the passage:

sell profits
products decrease
bean cost coffee shop either
increase price increase
or
coffee sales profits
will decrease decrease

We know for certain that if the bean cost goes up, then the shop’s profits will go down. Answer
(C) expresses this clearly.

(A) reverses the logic (profits decrease  bean cost increase).


(B) is very tempting, but it reverses the logic as well. Here’s what it says: profits decrease 
either sold noncoffee products or coffee sales decreased. But couldn’t there be some other reason
for a decrease in profits?
(D) is out of scope (we know nothing about what would happen if bean costs decreased).
(E) is out of scope. It speculates about what would happen if bean costs increase (which we
know something about), but also about what would happen if they don’t (we know nothing about
what might happen if they don’t).

23. (D)
Question Type: Identify the Flaw
The author assumes that if promises and expressions of good intentions are made in order to get
elected that they CANNOT at the same time be reliable. This isn’t necessarily true, is it? Imagine
a politician who says: “I promise to fight crime.” Perhaps she knows that this promise will help
her get elected, but perhaps the promise is actually one she plans to act on when elected. The two
aren’t necessarily mutually exclusive. Answer (D) expresses the fact that the author overlooks
this possibility.

(A) is tempting but incorrect. The argument does NOT make this presumption. In fact, the author
assumes the exact reverse of this statement.
(B) is out of scope (never kept by anyone under any circumstance?).
(C) is incorrect because this argument does not use any cause and effect relationship.
(E) is out of scope (worthiness is not at issue – the issue is whether the promises are reliable).

24. (E)
Question Type: Strengthen the Conclusion
The sociologist implies that evil people join institutions, thereby making the institutions
themselves evil. So, evil people come first, and then evil institutions follow. This argument
would break down if we learned that the nature of the institution actually determines the
characteristics of those who join it. Answer (E) strengthens the argument by striking down this
possibility.

(A) discusses doing good and evil, but the issue is really whether the people themselves are evil
to begin with.
(B) is a detail distortion (from good and evil to perfect and imperfect).
(C) is out scope (the issue is not whether people should or shouldn’t be optimistic, but rather
whether people are inherently evil).
(D) fails to get at the issue: do inherently evil people make institutions evil or do institutions
make people evil?

25. (A)
Question Type: Identify the Flaw
The argument made by some anthropologists is that if humans survived then we can conclude
that they had the ability to adapt to diverse environments:

survival (sufficient condition)  ability to adapt (necessary condition)

The author makes a suspect conclusion: the anthropologists are wrong because one case exists in
which a species had the ability to adapt but did not survive. In other words, the author assumes
that “survival  ability to adapt” implies “ability to adapt  survival.” The author has reversed
the logic and confused the sufficient condition with the necessary condition. Ability to adapt is
NOT sufficient to cause survival, as a species may have any one of many other shortcomings that
could lead to extinction. Answer (A) expresses this flaw in logic.

This is a confusing argument, so let’s try an analogous argument that contains the same flaw:

Many people say that because John lives in San Francisco he therefore lives in California. But
these people are wrong because I know someone who lives in California who does not live in
San Francisco.
In this argument, San Francisco is sufficient to require California (someone living in San
Francisco MUST live in California), but living in California is NOT sufficient to require San
Francisco, is it? It would be an error to assume that it does. The author of the argument makes a
similar mistake.

(B) is not an error made by the argument.


(C) is not an error made by the argument. A generalization is not made from a specific case.
(D) is very tempting. It is incorrect, though, because the author doesn’t necessarily fail to
consider that the species may have had some characteristics that lessened its chances of survival.
Rather, the author errs in concluding, regardless of what he or she believes about the species’
other characteristics, that the species’ ability to adapt was in itself sufficient to guarantee
survival.
(E) is in incorrect because it introduces causality. This argument does not introduce any
cause/effect relationships. Remember, conditional relationships are NOT necessarily causal
relationships.
Section IV: Reading Comprehension

Passage 1

This passage begins by providing background information and then introducing the first side of
the argument (that poetry and fiction should be treated as separate genres). However, the author
quickly rebuts this argument, and then uses the remainder of the passage to introduce and
describe an example (Rita Dove) that supports the second side of the argument (that poetry and
fiction can be mixed).

Summary of Paragraphs

Paragraph 1 starts by providing background information on the historical rift between poetry
and fiction in the United States, and then concludes by introducing the first side of the argument:
poetry and fiction should be kept separate.

Paragraph 2 attempts to explain why writers of poetry and fiction tend to support this rift
between the two genres.

Paragraph 3 gives the other side of the argument, that poetry and fiction can be successfully
mixed. The word “fortunately” gives a clear sense of what the author believes.

Paragraph 4 describes the work of Rita Dove and her ability to successfully mix the genres.

separation of
poetry and

-poets poetry and


-fiction writers fiction mixed
-US culture

-AUTHOR
-Rita
1. (E)
Question Type: Synthesis (1-2, 21-22, 24-26, 54-57)
Very rarely will the main point be stated explicitly in any one given place in the passage.
Questions that ask about the main point of the passage require us to synthesize multiple parts of
the passage in order to come away with a broader overall interpretation. Answer choice (E) does
this effectively. Notice that (E) also matches up nicely with our scale image.

(A) is an unsupported interpretation of the text. The passage states that Rita Dove is highly
acclaimed for her mixture of poetry and fiction, not just her ability to write lyrical fiction.
Furthermore, the scope of (A) is too narrow to cover the overall main point of the passage.
(B) is true, but its scope is too narrow to cover the overall main point of the passage.
(C) is attractive, but again its scope is too narrow.
(D) is an unsupported interpretation. Rita Dove mixes poetry and fiction, but this doesn’t
necessarily mean that she has abandoned the traditional techniques associated with the genres.

2. (D)
Question Type: Synthesis (24 -26, 54-57)
This question requires that we understand the nature of Rita Dove’s work so that we can then
move beyond the text to apply our understanding to a novel situation. Dove’s achievements are
defined by her ability to take two distinct genres (poetry and fiction) and blend them together,
despite the traditional separation of the two genres. Thus, our answer choice should provide an
example of someone who defies convention to mix two styles in a similar way. Answer (D) does
this.

(A) is an unsupported interpretation. Rita Dove did not mix a traditional style with an
untraditional style.
(B) is an unsupported interpretation. Rita Dove did move from one role to another.
(C) is an unsupported interpretation. Rita Dove did not team up with anyone else.
(E) is an unsupported interpretation. Rita Dove did not add to an already existing work.

3. (A)
Question Type: Identification (8-11)
This question requires that we simply identify supporting text in the passage. If we understand
the structure of the passage, the language in the question, “widely held view,” should lead us
directly to the first paragraph. Remember, this is where the first side of the argument was
introduced (the conventional view that poetry and fiction should be kept separate. In lines 8-11,
we see that there is a widely held view that character and narratives should be left to fiction.
Answer choice (A) is correct.

(B) is an unsupported interpretation.


(C) is a contradictory interpretation. The passage states that graduate writing programs exist in
both poetry and fiction.
(D) is a contradictory interpretation. The conventional view is that lyrical language should stay
in poetry and NOT be mixed with fiction. Know the sides of your scale!
(E) is a contradictory interpretation. European literary cultures (at least the culture in Germany)
emphasize a more generalist approach.

4. (E)
Question Type: Inference (21-22)
This question requires that we use information in the text to make an inference about the author’s
attitude towards the conventional rift between poetry and fiction. The word “fortunately” in line
21 is a good clue that the author is against this rift, and is glad that there are emerging examples
of writers who are mixing the two genres. If you got your scale right to start, this question should
be straightforward. Answer (E) is the correct answer.

(A) is a contradictory interpretation. The author is not perplexed as to what could have caused
the rift. In fact, paragraph 2 explains what caused the rift.
(B) is a contradictory interpretation. Academics have not overlooked the rift. In fact, they
support it.
(C) is an unsupported interpretation. The author is not ambivalent. In fact, he/she is very much
on one side of this issue.
(D) is a contradictory interpretation. The author believes that the rift is being overcome (by
writers such as Rita Dove).

5. (D)
Question Type: Identification (15-17)
This question requires that we simply identify supporting text in the passage. If we have a solid
understanding of the passage structure, the language in the question (“the cause of the deep rift”)
should lead us directly to paragraph 2. It is the suspicion of the generalist that has led to the rift,
and answer choice (D) correctly expresses this.

(A) is an unsupported interpretation. Nowhere does the passage indicate an attitude of superiority
coming from poets or fiction writers.
(B) is an unsupported interpretation. We have no information about the differences in the way
graduate programs train poets versus fiction writers.
(C) is out of scope. Where did “publishers” come from?
(E) is out of scope. The degree to which fiction is read versus poetry is not covered in the
passage.

6. (B)
Question Type: Inference (24-36)
This question requires us to understand the function of paragraph 3. This paragraph introduces
Rita Dove as an example of a writer who has successfully mixed poetry and fiction (in order to
support the author’s view that there is no need for the rigid separation). Remember that just
about everything in the passage will be used directly or indirectly to support one side of the scale
or the other. We want to choose an answer that interprets the text in lines 32-36 in this way.
Answer choice (B) does this. This ultimate purpose of this piece of text is support the notion that
poetry and fiction should not be rigidly separated.
(A) is out of scope. Nowhere are English-speaking societies compared with other societies.
(C) is an unsupported interpretation. It misses the point.
(D) is an unsupported interpretation. Again, this fails to get at the real reason why this part of the
text has been included.
(E) is a bit more attractive because it does reference an opposition to the separation of fiction and
poetry, but nowhere do we get any indication that the Germany experience was the origin of her
opposition. This is an unsupported interpretation.

7. (A)
Question Type: Inference (42-54)
In the final paragraph, the author describes Dove’s work in very specific terms, first describing
Dove’s poetry and how she has managed to weave in elements of narrative fiction. Next the
author discusses Dove’s fiction and her ability to weave in elements of lyric poetry. While it
doesn’t state this explicitly, we can infer that Dove works primarily in one genre or the other, but
introduces both styles in each of her works. Answer choice (A) expresses this inference.

(B) is a contradictory interpretation. In fact, the passage states that lyric narrative “evokes
emotion and inner states without requiring the reader to organize ideas or events in a particular
linear structure.”
(C) is an unsupported interpretation. We don’t have any evidence to suggest that she was the
first.
(D) is an unsupported interpretation. A relative comparison between lyric narrative and pure lyric
poetry is not made.
(E) is an unsupported interpretation. While the passage does indicate that German writers write
in many different styles (including drama), we can’t infer that writers who cross the generic
boundary between poetry and fiction generally try writing dramatic pieces. This is too big of a
leap, and there is no support for this interpretation.

8. (A)
Question Type: Synthesis (1-2, 21-24)
This question requires that we really understand the overall flow of the passage (in other words,
our understanding of the scale is very important). In summary, there has historically been a deep
rift between poetry and fiction, but recently there are signs that this rift is diminishing. Rita Dove
is an example. If we were to predict the future, we could justifiably assume that the future might
see a growing number of writers crossing the generic boundaries between genres. Answer (A)
expresses just this.

(B) is out of scope. The passage mentions nothing of the market for lyric poetry.
(C) is an unsupported interpretation.
(D) is a contradictory interpretation. This is the exact opposite of what the passage indicates.
(E) is out of scope. We know nothing of the size or nature of the reading audiences of poetry or
fiction.
Passage 2 (Comparative Passages)

The evolutionary advantage (or lack thereof) of humans’ ability to make music is the central
argument contained in this set of comparative passages. The author of Passage A concludes that
the ability to make music had little adaptive value to humans while the author of Passage B
makes an argument for music’s adaptive value.

Summary of Paragraphs

Passage A, Paragraph 1 introduces a comparison between language and music.

Passage A, Paragraph 2 starts by giving background information on the neurological


similarities between language and music, but then notes one difference between humans’
language and musical abilities: in general, people are better at language than they are at music.

Passage A, Paragraph 3 uses the end of paragraph 2 to come to the conclusion that language
was the primary function that natural selection operated on, and that music was simply a
byproduct with limited utility.

Passage B, Paragraph 1 starts by quoting Darwin. Darwin believed music had no adaptive
value. The author quickly counters, and we begin to get a sense for the central argument.

Passage B, Paragraph 2 gives background information on mother-infant interactions in order to


show the important role that music plays in child rearing.

Passage B, Paragraph 3 uses the evidence in paragraph 2 in order to emphasize the


evolutionary advantage that music confers.

M as a M has adaptive
byproduct of L value on its own

-AUTHOR A -AUTHOR B
-Darwin (-mother/infant
(-humans better at interaction)
language than
9. (C)
Question Type: Synthesis (1-2, 19-27, 32-37, 57-62)
To answer this question correctly, we must have a strong understanding of the scale (the central
argument). Both of these passages aim to uncover the evolutionary advantages (or lack thereof)
of humans’ ability to make music. Answer (C) comes closest to expressing this.

(A) is narrow in scope. While both passages mention brain size, it is not the central focus of
either passage.
(B) is a half-scope answer. Mother-infant bonding is discussed in passage B but not passage A.
(D) is a half-scope answer. A comparison of music and language is made in passage A but not in
passage B.
(E) is a half-scope answer as well. A comparison of music and language is made in passage A
but not in passage B.

10. (B)
Question Type: Identification (3, 35)
This question requires that we simply identify supporting text in each of the two passages. It’s
basically a hunting expedition. Both passages mention music and its impact on human emotion.
On a question like this, be sure to find the proof text in the passage(s).

(A) is a half-scope answer. Bonding between humans is mentioned in passage B but not passage
A.
(C) is a half-scope answer. Passage A mentions “brain imaging studies” (neurological research),
but passage B never discusses neurological research per say. It does discuss the “neurological
basis” of music, but not neurological research.
(D) is a half-scope answer. The helplessness of hominid infants is mentioned in passage B but
not in passage A.
(E) is a half-scope answer. The use of tools to produce sound is mentioned in passage A but not
passage B.

11. (D)
Question Type: Synthesis (21-27, 32-37)
This question is framed as an inference question, but it requires more than an inference. This
question requires that we understand the central argument and where each author stands in
relation to the central argument. Notice that answer (D) expresses our scale almost exactly. The
author of passage B would agree with answer (D) while the author of passage A would disagree.

(A) is a half-scope answer. Premature births were discussed in passage B but not passage A.
(B) is a half-scope answer. Music and language are compared in passage A but not passage B.
(C) is a contradictory interpretation. The authors actually agree on this point! Remember, we’re
looking for a point on which they disagree.
(E) is a half-scope answer. Passage B discusses mother-infant bonding but passage A does not.

12. (C)
Question Type: Inference (19-21, 51-62)
Both passages discuss an increase in brain size and its relation to the evolution of music ability.
Notice the soft language in answer (C): “at least partly a result of evolutionary increases in brain
size.” It’s very hard to argue with an answer choice written with this sort of language.

(A) is a half-scope answer. Passage B discusses music and early infancy while passage A does
not.
(B) is a half-scope answer. Passage B discusses mother-infant bonds but passage A does not.
(D) is an unsupported interpretation. Both passages discuss the neurological basis of humans’
ability to make music (passage A more than passage B), but neither discusses the development of
new neurological systems.
(E) is a half-scope answer. Passage A discusses the differences in musical ability among
different people but passage B does not. Furthermore, neither passage discusses why ability
differs so greatly.

13. (E)
Question Type: Inference (21-24, 32-37)
Passage A states “the primacy of language over music that we can observe today suggests that
language, not music, was the primary function natural selection operated on.” Passage B cites
mother-infant bonding rituals, rituals that we observe to this day. In either case, the author uses
modern human behavior to draw a conclusion about the evolutionary origins of music. Answer
(E) expresses this.

(A) is out of scope. Nonhuman animals are not discussed.


(B) has a degree issue. “All” human capacities? This goes too far.
(C) is a half-scope answer. Passage A discusses the one neurological system that underlies two
capacities (music and language), but passage B does not.
(D) is an unsupported interpretation. Nowhere does either passage argue that if you discover the
neurological basis of a behavior then you have thereby discovered the essence of the behavior. In
fact, passage A places the capacity to make music in the same neurological basis as the capacity
to make music, but draws different conclusions about the importance of music vs. language.

14. (A)
Question Type: Synthesis (19-27, 32-37)
If we have a solid understanding of the central argument, we know that the two authors draw
different conclusions. (A) is exactly right. Passage A uses music in comparison with language in
order to conclude that music had little adaptive value. Passage B uses mother-infant interactions
to conclude that music did have adaptive value. This is our scale!

(B) is incorrect, but tempting. Passage A does pose a question to start, but it is not a question that
passage B attempts to answer. In fact, passage B doesn’t compare music and language.
(C) is incorrect because it puts the two passages on the same side.
(D) is incorrect because there is no indication that passage A expresses a stronger commitment
than passage B.

(E) is incorrect because it puts the two passages on the same side.
Passage 3

The central argument (strengthen copyright laws to protect online copyright holders vs. do not
strengthen laws) is introduced very early on in this passage. The entire final paragraph is
dedicated to clarifying the author’s position.

Summary of Paragraphs

Paragraph 1 starts by providing background information on the World Wide Web, but then
introduces the central argument of the passage.

Paragraph 2 informs the debate by describing the unique copyright issue that exists for online
content. It finishes by posing a question that serves as a springboard for the final paragraph.

Paragraph 3 answers the question posed at the end of paragraph 2. Paragraph 3 makes the
author’s position very clear.

strengthen
copyright laws

do not strengthen
-intellectual copyright laws
property owners

-AUTHOR
-web users
15. (A)
Question Type: Synthesis (20-27, 28-30, 40-44)
From our scale image, we would conclude that the main point is that copyright laws should NOT
be strengthened. While answer (A) isn’t an exact representation of this, it does accurately express
the author’s viewpoint. The author does NOT believe that linking to a document should be
considered copyright infringement.

(B) is very tempting, but it is an unsupported interpretation. The first half of this answer matches
exactly with our scale. However, answer (B) goes off the tracks with “unless such changes
amplify rather than restrict the free exchange of ideas…” We might assume that the author
believes this (lines 52-55), but he/she never actually states this.
(C) is a contradictory interpretation. The author actually says that preventing access impedes the
free exchange of ideas (49-50). Furthermore, this answer is too narrow to cover the main point.
(D) is an unsupported interpretation. It misses the point of the passage.
(E) is tempting, but it has some degree problems. It’s true – the author favors the free exchange
of ideas over a strengthening of copyright laws, but notice the modifiers in this answer choice:
far outweigh, small number of individuals, radical alteration. This answer choice is too extreme.

16. (A)
Question Type: Inference (6-9)
The owners of intellectual property would want to restrict the use of that property by others.
Thus, strengthening copyright laws would mean making these laws more restrictive.

(B) is an unsupported interpretation.


(C) is an unsupported interpretation.
(D) is an unsupported interpretation.
(E) is an unsupported interpretation.

17. (E)
Question Type: Inference (44-50)
In lines 44-50, the author acknowledges that password-protected pages would inhibit the free
exchange of web content. Thus, we can infer that the author believes unrestricted flow of content
cannot be achieved without complete and unrestricted access. Answer (E) expresses this. Notice
that this inference is not a big leap. It’s not explicitly stated in the text, but it is one small baby
step from the information given in the text.

(A) is a contradictory interpretation. The author is against the strengthening of laws.


(B) has a degree issue. The author says that the free exchange of ideas would be compromised
“somewhat,” not “significantly diminished.”
(C) is a contradictory interpretation. The author actually states that intellectual property owners
can restrict access through password protection.
(D) is a contradictory interpretation. The author does not want to alter legal codes.

18. (C)
Question Type: Synthesis (49-55)
In the last paragraph, the author says that content owners can protect their content if they choose
to (through password protected pages), and that this option of self-protection, even if it might
slightly inhibit the free exchange of content, would be better than a broader strengthening of
laws, which would severely limit the free exchange of ideas. We are asked to take this
understanding and apply it to an outside analogy. In this case, we’re looking for something that
presents two options: severe limitation imposed at a broader level vs. moderate limitation that is
self-imposed. Answer (C) expresses this relationship.

(A) is an unsupported interpretation.


(B) is an unsupported interpretation.
(D) is an unsupported interpretation.
(E) is an unsupported interpretation.
19. (B)
Question Type: Inference (40-44)
In paragraph 3, the author states that person A posts the document (thereby making it accessible)
and “controls access to the document,” while person B simply links to the document. Then the
author says that creating a link is not the same as distributing the document. Thus, we can infer
that the author believes person A to be in control of distribution, not person B.

(A) is out of scope. The passage does not discuss the possibility of being a linker and an owner
of content simultaneously. Furthermore, to say definitively that there are “no” people who fall
into this category would be a very extreme statement.
(C) is out of scope. Access is discussed, by not privacy rights.
(D) is out of scope. This is subtle. The issue discussed isn’t who has primary control over who
reads the document, but rather who has primary control over distribution of the document. These
are two very different things. The former is not addressed.
(E) is out of scope.

20. (E)
Question Type: Inference (36-44)
The answering machine greeting is equated with a document posted on the web. By posting the
greeting, or the document, the owner has made it publicly available. The author uses this analogy
to make the case that the owner of the content is actually in control of the distribution of that
content, and that by posting the message, or posting the document, the owner is in fact offering it
up for distribution.

(A) is out of scope. The type of medium is irrelevant.


(B) is an unsupported interpretation.
(C) is out of scope. The speed of transmission is irrelevant.
(D) is out of scope. Whether or not people care is beside the point.

21. (D)
Question Type: Synthesis (6-13, 23-27, 28-55)
The author’s position is: linking to a web page is not copyright infringement, so we shouldn’t
strengthen laws to protect against this. The answering machine analogy supports this position by
clarifying who is responsible for the actual distribution of content. This issue of distribution is
the basic principle at work.

(A) is an unsupported interpretation. The answering machine is not compared with the web, but
rather used as an example to support a claim made about the web.
(B) is an unsupported interpretation. The answering machine is used as an analogy to support the
author’s side, but it is not used to illustrate the position of the other side.
(C) is an unsupported interpretation. This is not the point at all.
(E) is an unsupported interpretation. Again, this is not the point.

22. (D)
Question Type: Identification (20-23)
This fact about current copyright law is mentioned in the second paragraph. If we have a solid
understanding of the passage structure, we know where to look.

(A) is a contradictory interpretation. “Completely unrestricted use” is not right. In fact, the
passage implies the opposite: current laws prohibit the distribution of unauthorized copies of
documents.
(B) is an incorrect interpretation. The passage says that owners of the content control
distribution.
(C) is out of scope. Profit is not discussed.
(E) is a contradictory interpretation. The author believes the opposite.

Passage 4

This passage compares two approaches to tracing the history of the Irish landscape. While this
passage doesn’t really set up sides of a central argument, we can place the two central concepts
in opposition to each other as we read: using historical documents vs. using fossilized pollen
grains. The author’s point is that the use of fossilized pollen grains gives a richer picture of the
history of the Irish landscape than do historical documents alone.

Summary of Paragraphs

Paragraph 1 introduces a traditional approach (using historical documents) for tracing the
evolving Irish landscape, but then discusses a few of the shortcomings of this approach. If we are
reading actively, we can anticipate that a different, better approach will be described next.

Paragraph 2 introduces an additional, supplementary method (using fossilized pollen grains) for
tracing the landscape.

Paragraph 3 provides an example of how the fossilized pollen method has corrected, or
supplemented, a traditional understanding of the landscape.

Paragraph 4 gives another such example.

Paragraph 5 discusses one major shortcoming of the pollen grain approach.

primarily historical
documents
to trace landscape

fossilized pollen
grains in addition to
scholars historical documents

AUTHOR
-cereal
-flax
23. (A)
Question Type: Synthesis (1-3, 18-20, 30-32, 44-47)
Answer (A) is basically an exact representation of our scale understanding.

(B) is very tempting, but it is an unsupported interpretation. The historical documents did not
help to revise previously accepted hypotheses. In fact, the historical documents were used to
create the previously accepted hypotheses. It was the pollen record that led to the revision.
(C) is a contradictory interpretation. Paragraph 4 clearly states that fossilized pollen does NOT
help in identifying plant species.
(D) is a contradictory interpretation. Paragraph 4 clearly states that fossilized pollen is not able to
distinguish madder from other plants.
(E) has a degree issue. “Severely limited” is too strong, especially considering all the evidence
given for the benefits of using the pollen record.

24. (B)
Question Type: Identification (30-32)
This question is tricky. Remember, we want an answer that presents an old view that has been
revised by the pollen record. Historical evidence seemed to indicate that cereal grains were not
cultivated until the seventh century, but the pollen record indicates otherwise.

(A) is a contradictory interpretation. The passage doesn’t provide evidence against this fact, but
rather states that it is true.
(C) is an unsupported interpretation. The passage doesn’t question whether or not cereal grain
has been cultivated “continuously,” but rather when it was first cultivated.
(D) is a contradictory interpretation. The passage doesn’t provide evidence against this fact, but
rather states that it is true.
(E) is a contradictory interpretation. The passage doesn’t provide evidence against this fact, but
rather states that it is true.

25. (D)
Question Type: Identification (5-9)
The end of paragraph 1 gives more information on the “historical documents,” also referred to as
the “documentary record” in lines 20 and 37. These documents related to military or commercial
interests. Answer (D) expresses this.

(A) is an unsupported interpretation. The documentary record refers to the old historical
documents, NOT the pollen record.
(B) is an unsupported interpretation. The documentary record refers to the old historical
documents, NOT the pollen record.
(C) is an unsupported interpretation. Current historians use these documents, but these current
historians did not write these documents.
(E) is an incorrect interpretation. Current historians use these documents, but these current
historians did not write these documents.
26. (E)
Question Type: Identification (42-47)
The third paragraph discusses flax cultivation and how the pollen record revised a previous
understanding of when flax cultivation began.

(A) is out of scope. Flooding is not discussed.


(B) has degree issues. Historians believed that cereal grain cultivation had not occurred in the
Long Lough region of County Down to any significant extent before the seventh century (because
of the soil there), not that it hadn’t occurred anywhere in Ireland.
(C) is an unsupported interpretation. We don’t know that historians thought the history was “well
documented.”
(D) is an unsupported interpretation. We don’t have any evidence for this.

27. (C)
Question Type: Synthesis (21-32, 48-58)
Without looking at the answers, we know that the second paragraph provides an example of the
utility of the pollen record while the final paragraph discusses a shortcoming of the pollen
record. In essence, paragraph 2 says “here’s an example of how the pollen record can help,” and
paragraph 4 says “but the method has its limitations.” So, paragraph 4 qualifies the claim made
in paragraph 2.

(A) is a contradictory interpretation. The final paragraph does NOT support the second
paragraph. In fact, it may even weaken it.
(B) is an unsupported interpretation. Paragraph 4 does introduce a shortcoming of the method,
but this shortcoming doesn’t necessarily need to be solved or fixed before the method can be
used.
(D) is an unsupported interpretation.
(E) is an unsupported interpretation. The final paragraph does not offer procedures.

You might also like